User Avatar
eguerrero192
Joined
Apr 2025
Subscription
Free
PrepTests ·
PT101.S2.Q21
User Avatar
eguerrero192
Monday, Aug 31 2020

Hang on so, if we get a premise like All kittens are effective, there are 2 ways to map it:

1) K --> eff. (If it's a kitten it's effective)

OR

2) K eff

For this Q, JY opted for the second version, but how do we know when to use that version?

#help (Added by Admin)

PrepTests ·
PT104.S1.Q19
User Avatar
eguerrero192
Saturday, Aug 29 2020

If you read and understand E, it's the clearest and most obvious answer choice. (Maybe it only seems like it after you do the Q; but maybe not)

PrepTests ·
PT103.S2.Q19
User Avatar
eguerrero192
Saturday, Aug 29 2020

This question is flawed in my opinion. Although I did get the answer right, I don't think you can infer E from the passage. The passage says that violent behaviour in response to unpleasant stimuli is a product of one's cultural environment - but what about violent behaviour that is in response to PLEASANT stimuli or OTHER types that are not unpleasant? By claiming that E is inferable from the passage, you're assuming that all violent behaviour results in response to unfamiliar stimuli.

PrepTests ·
PT155.S2.Q26
User Avatar
eguerrero192
Sunday, Sep 27 2020

For me, it looks like the trick in this question was the subtle and sneaky use of referential phrasing (or maybe it wasn’t so sneaky and it’s just my brain activity declining at 1AM)

“Since X, this indicates that...”

This is referential phrasing, referring to the claim that lake stickleback have no armour. So it’s saying:

Lake SBs have no armour to fend off predators. Since X is the case, this (the fact that lake SBs have no armour to defend against predators) must be because having a larger size (being bigger) is a better defence against predators than having armour.

Assumption: there isn’t some other reason why they have no armour. There’s no other reason or explanation for this phenomenon (of an absence of armour) besides this apparent evolutionary hypothesis. Maybe they have no armour because having a larger size helps them in some other way (and/or, like JY said, there ARE no predators near the lake SB).

PrepTests ·
PT155.S4.Q22
User Avatar
eguerrero192
Sunday, Sep 27 2020

Because the whole, to which a particular part belongs, is X, that part is also X.

(A) is correct. Because the system of interconnected lakes (of which X lake is a part of) is one of the most beautiful of its type, Wooded Lake is one of the most beautiful of its type.

PrepTests ·
PT123.S2.Q22
User Avatar
eguerrero192
Sunday, Sep 27 2020

Here’s a better explanation: A is wrong because it says “likely” rather than “more likely”.

If it said more likely, it’d probably be the CA, given that D is not really a supported AC due to a subtle ambiguity. The only source of discouragement mentioned, technically speaking, is the isolation. The passage mentions two contributing factors to that phenomenon/source, but that doesn’t mean those two factors are each individual sources.

PrepTests ·
PT123.S2.Q11
User Avatar
eguerrero192
Saturday, Sep 26 2020

“Unwarranted” is the key word here that makes D require an inference larger than the one required for C to be true.

Note, the conclusion has the word “probably”. So it’s certainly a stretch to infer that this conclusion statement means it is unwarranted

PrepTests ·
PT123.S2.Q8
User Avatar
eguerrero192
Saturday, Sep 26 2020

Does “will have worse effects than proponents may believe” = “may have worse effects than proponents (do) believe

Because that’s the only thing that I can see justifies this AC

PrepTests ·
PT124.S3.Q25
User Avatar
eguerrero192
Friday, Sep 25 2020

C is literally a virtual rewording of “none of the issues at the meeting are relevant to terry”

PrepTests ·
PT111.S1.Q20
User Avatar
eguerrero192
Monday, Sep 21 2020

I tried hard to understand this Question despite getting it right. It seems that many explanations in below comments apply strategies that don’t work in other questions like using lawgic to bring the premise down into the conclusion. I was frustrated also at JY’s explanation at first but now believe it does a more accurate job explaining the Q than any of the comments I’ve seen. This is how I see it

Quite simply this would be the lawgic

NOG

MTC → E

———————-

AA→ E

The SA then is: Since [anyone who manages to convince → expert], [almost anyone → expert]. (Read only the words written)

You can’t diagram this resulting sufficient assumption in terms of lawgic. That’s the problem. But another way of phrasing this sufficient assumption is: anyone who manages to convince, is almost anyone (is equal to almost anyone).

The author is arguing (rightly or wrongly) that because anyone who MTC can X, that must mean that almost anyone can X.

Anyone who hears this argument intuitively thinks: ok so it sounds like you’re assuming that: anybody who manages to convince = almost anyone. Assuming that: because anybody who MTC is X, almost anyone is X. By saying “this is true for X because it’s true for Y”, you’re assuming X and Y to be at least fundamentally similar in one sense. If I say: anyone who skateboards is good-looking, therefore almost anyone is good looking, I’m assuming that almost anyone skateboards (well, at least, this is an assumption SUFFICIENT for my conclusion)

S—> GL (Standing in for MTC —> E)

AA —> S (SA)

—————-

AA —> GL

The SA makes the argument valid.

The fact that you’re deriving your conclusion this way literally means you’re assuming if you’re anybody who MTC, then you’re almost anybody; or how (A) puts it, if you’re almost anybody then you can MTC. This isn’t reversing the logical structure despite it seeming like it is, bc the terms are taken to be equal. A is B and B is A.

Even if you ignore all this, don’t get upset with Qs that seem invincible to your strategies, the vast majority of them compensate by making the WAs pretty obviously wrong

PrepTests ·
PT106.S1.Q20
User Avatar
eguerrero192
Sunday, Sep 20 2020

For those that had trouble with this Q at first (like myself): you can think of (A) alternatively as saying: It is always the case that either the economy is weak or investment decreases. We know this must be false because we know by virtue of the contra positive that if investment does not decrease, the economy is not weak. We know that investment does not decrease, so we know for certain that the economy is not weak. Therefore, the current situation is one where investment does not decrease AND the economy is not weak; or in other words, a situation where neither investment decreases NOR the economy is weak.

So it must be false that it is always the case that either the economy is weak or investment decreases.

As far as I can tell (though I could be wrong), the matter of whether the “or” is exclusive is irrelevant to the AC in this case. Regardless of whether it’s possible that BOTH the economy is weak AND a investment decreases, the thing that makes (A) a MBF AC is that you can (and indeed do, per the last premise) have a situation where NEITHER occurs.

PrepTests ·
PT148.S3.Q23
User Avatar
eguerrero192
Tuesday, Oct 20 2020

So now there are strengthen questions without premises. Fuck this test I’m out. The new prep tests are genetically modified mutations engineered to use the full extent of psychometric data to screw you

PrepTests ·
PT101.S2.Q7
User Avatar
eguerrero192
Tuesday, Sep 15 2020

Again, I have to call out a discrepancy here between the way this Q requires you to approach it, and the whole “never attack the premises or conclusion” structure taught in the lessons. How would, C, for example, be weakening the support structure? The necessary assumption you need to draw from it to relate it to the stimulus is a statement that contradicts the premise. The Only mechanism by which it weakens the argument is by undermining the premise that access to a recreation centre is a necessity for people in the neighborhood. Am I missing something?

If we’re assuming/conceding the truth of the premise, how does C still weaken the argument?

#help (Added by Admin)

PrepTests ·
PT103.S3.Q6
User Avatar
eguerrero192
Tuesday, Sep 15 2020

Assumption: The consideration of gender-balance in (/the gender of) the patients base on which data is collected is irrelevant to science.

CA: Restricting the gender of this patients base to only males can lead to inaccurate results. [Therefore, the consideration of gender-balance in the patients base on which data is collected IS relevant to science.

PrepTests ·
PT102.S4.Q3
User Avatar
eguerrero192
Tuesday, Sep 15 2020

Am I missing something? Isn’t the obvious assumption that the # of ppl working in the nuclear plant is the same as the # of ppl working in other industrial plants, only on those grounds rendering the conclusion warranted?

#help (Added by Admin)

User Avatar
eguerrero192
Wednesday, Oct 14 2020

@ said:

I'm pretty sure it becomes available the same time as it does for everybody on Oct 23

Why would it be Oct 23? What about those who took it on October 8th? LSAC says it takes minimum 2 weeks; but that's rare

PrepTests ·
PT148.S1.Q17
User Avatar
eguerrero192
Tuesday, Oct 06 2020

How is it made clear that they're speaking about the same lake, and not some other bay or body of water?

PrepTests ·
PT147.S4.Q10
User Avatar
eguerrero192
Tuesday, Oct 06 2020

Even without lawgic, you can tell from the context of the statement that the author is offering a counter example in support (as the premise) of the conclusion that it is not the case that X —> Y.

Once you recognize it’s intended as a counter example, you know that what the premise is trying to get at (or what it needs to get at to make the argument valid) is the idea that it’s possible you can have [EV INC and NOT CALIBRE DEC]. You can’t write two whole new premises in a SA Q, so you’ll need to realize that “the most entertaining” is meant to stand in for EV incl. which means, turning to the second part of the counter-example premise given, the sufficient assumption MUST be some way of equating “the greatest journalists” with [NOT calibre dec].

PrepTests ·
PT146.S1.Q14
User Avatar
eguerrero192
Monday, Oct 05 2020

The AC here would be likely best reached by

(1)prephrasing the assumption as one that rests in the similarity between the 2 events in the principle and the 2 events in the counter example. And since the second event (2nd term in the CS) is the same across principle/CE; prephrase that the assumption must be related to the assumed similarity between the first term/event in each: that doing what a person promised to do is ‘something that one ought to do’.

(2) using PoE, mainly by recognizing A is wrong bc it offers the opposite (even when contraposed) of what we need, and bc it is a cookie cutter wrong AC in that it overgeneralizes beyond what is required for the arg.

To check D is correct, the negation test reads: If the promise to do X cannot be kept (is impossible to keep), then doing what one promised (doing X) is not something that one has an obligation to do (not something that one “ought to do”).

Bingo. The counter example loses relevance as support for the conclusion (it falls apart). If doing what one promised is not an instance of what one ‘ought to do’, the CE falls apart as a contradiction of the principle.

PrepTests ·
PT146.S1.Q17
User Avatar
eguerrero192
Monday, Oct 05 2020

The main reason C is wrong is because you can upgrade the stoplights without adding new ones. Say you just replaced the old ones. Doesn’t help explain jack shit lol. Realized this in BR.

PrepTests ·
PT102.S3.Q7
User Avatar
eguerrero192
Friday, Sep 04 2020

MoR: Percentage vs Quantity

Claims: Group A is 'more prone to causing accidents' than Group B;

which means the proportion of Group A that is X is greater than the proportion of Group B that is X

Flaw: BUT: Provides no evidence about the proportion, which is needed to support the argument. Instead, supports by saying the AMOUNT of accidents caused by (/the amount of X assigned to) Group B is greater than the AMOUNT of accidents caused by Group A. Even if this is true (let's also ignore the blatant sample-size error it's making), it doesn't support the claim that is fundamentally about proportion.

If 10 accidents are caused by group A, and 50 accidents are caused by group B, that says nothing about which is more PRONE to causing accidents; for that, you'd need to know proportions: Out of Group B, what proportion of people causes accidents?

IOW: The group with the higher proportion of X is more prone, NOT the group with the higher amount (occurences) of X.

PrepTests ·
PT142.S1.Q23
User Avatar
eguerrero192
Saturday, Oct 03 2020

Copy pasted from a user below:

so thinking about this –the argument jumps from “plants and animals” to “living organisms” well couldn’t a living organism be super teeny bacteria or even a virus? We aren’t given a definition.

If there are bacteria inside the earth’s crust, maybe that’s why there are bio markers in petro! and if that’s the case then the theory that petro formed from deep carbon deposits could hold thereby WEAKNING the argument that petro formed from the fossils of plants/animals ~ answer choice (D)!

PrepTests ·
PT105.S2.Q19
User Avatar
eguerrero192
Thursday, Sep 03 2020

Are there any other Qs like this? #help

PrepTests ·
PT111.S4.Q23
User Avatar
eguerrero192
Thursday, Sep 03 2020

A and D both have the same logical structure.

However, D is correct because:

1) D is a comparative statement [implicitly] about two different subjects/entities, one subject compared to the other. A is a comparative statement about a comparison within ONE subject (X gets wiser than X was before, X gets older than X was before)

2)* In D, the subjects in the conclusion are related on a geographical scale, just like the subjects in the conclusion of the stimulus. Their relationship is geographical.

This idea is rough (and probably poorly communicated), but if anybody salvages something meaningful from it, great

PrepTests ·
PT109.S3.Q22
User Avatar
eguerrero192
Wednesday, Sep 02 2020

When are we supposed to use Lawgic for PMoR Qs, and when are we NOT supposed to?

#help (Added by Admin)

Confirm action

Are you sure?